K
Khách

Hãy nhập câu hỏi của bạn vào đây, nếu là tài khoản VIP, bạn sẽ được ưu tiên trả lời.

4 tháng 4 2017

C1:Biến đổi tương đương

\(\dfrac{1}{x}+\dfrac{1}{y}\ge\dfrac{4}{x+y}\Leftrightarrow\dfrac{x}{xy}+\dfrac{y}{xy}\ge\dfrac{4}{x+y}\)

\(\Leftrightarrow\dfrac{x+y}{xy}\ge\dfrac{4}{x+y}\Leftrightarrow\left(x+y\right)^2\ge4xy\)

\(\Leftrightarrow x^2+y^2+2xy\ge4xy\Leftrightarrow x^2+y^2-2xy\ge0\Leftrightarrow\left(x-y\right)^2\ge0\)

C2:Dùng AM-GM

\(x+y\ge2\sqrt{xy}\);\(\dfrac{1}{x}+\dfrac{1}{y}\ge2\sqrt{\dfrac{1}{x}\cdot\dfrac{1}{y}}=2\sqrt{\dfrac{1}{xy}}\)

Nhân theo vế 2 BĐT

\(\left(x+y\right)\left(\dfrac{1}{x}+\dfrac{1}{y}\right)\ge4\sqrt{xy\cdot\dfrac{1}{xy}}=4\Rightarrow\dfrac{1}{x}+\dfrac{1}{y}\ge\dfrac{4}{x+y}\)

C3:Dùng Cauchy-Schwarz (dạng Engel)

\(\dfrac{1}{x}+\dfrac{1}{y}\ge\dfrac{\left(1+1\right)^2}{x+y}=\dfrac{4}{x+y}\)

-3 cách trên đều có dấu "=" khi \(x=y\)

28 tháng 2 2022

\(\left(x+y+z\right)^2=x^2+y^2+z^2+2xy+2xz+2yz=z^2+\left(x+y\right)^2+2z\left(x+y\right)=36\)

áp dụng BĐT cosi : 

\(z^2+\left(x+y\right)^2\ge2z\left(x+y\right)\)

<=> \(z^2+\left(x+y\right)^2+2z\left(x+y\right)\ge4z\left(x+y\right)=36< =>z\left(x+y\right)\ge9\)

ta lại có \(\dfrac{x+y}{xyz}=\dfrac{x}{xyz}+\dfrac{y}{xyz}=\dfrac{1}{yz}+\dfrac{1}{xz}\) áp dụng BĐT buhihacopxki dạng phân thức => \(\dfrac{1}{yz}+\dfrac{1}{xz}\ge\dfrac{4}{yz+xz}=\dfrac{4}{z\left(x+y\right)}\ge\dfrac{4}{9}\left(đpcm\right)\)

dấu bằng xảy ra khi \(\left[{}\begin{matrix}yz=xz< =>x=y\\x+y+z=6\\z^2=\left(x+y\right)^2\end{matrix}\right.< =>\left[{}\begin{matrix}x+y+z=6\\z=2x=2y\end{matrix}\right.< =>\left[{}\begin{matrix}x=y=\dfrac{3}{2}\\z=3\end{matrix}\right.\)

28 tháng 2 2022

-Ủa vì sao\(\dfrac{4}{z\left(x+y\right)}\ge\dfrac{4}{9}\)? Đáng lẽ là \(\dfrac{4}{z\left(x+y\right)}\le\dfrac{4}{9}\) chứ?

AH
Akai Haruma
Giáo viên
26 tháng 2 2018

Lời giải:

Ta có:

\(A=\frac{1}{x^2+x}+\frac{1}{y^2+y}+\frac{1}{z^2+z}=\frac{1}{x(x+1)}+\frac{1}{y(y+1)}+\frac{1}{z(z+1)}\)

\(=\frac{1}{x}-\frac{1}{x+1}+\frac{1}{y}-\frac{1}{y+1}+\frac{1}{z}-\frac{1}{z+1}\)

\(=\left(\frac{1}{x}+\frac{1}{y}+\frac{1}{z}\right)-\left(\frac{1}{x+1}+\frac{1}{y+1}+\frac{1}{z+1}\right)(1)\)

Áp dụng BĐT Cauchy-Schwarz:

\(\frac{1}{x}+\frac{1}{1}\geq \frac{4}{x+1}\) và tương tự với các phân thức còn lại rồi cộng lại:

\(\Rightarrow \frac{1}{x}+\frac{1}{y}+\frac{1}{z}+3\geq 4\left(\frac{1}{x+1}+\frac{1}{y+1}+\frac{1}{z+1}\right)\)

\(\Leftrightarrow \frac{1}{x+1}+\frac{1}{y+1}+\frac{1}{z+1}\leq \frac{1}{4}\left(\frac{1}{x}+\frac{1}{y}+\frac{1}{z}+3\right)(2)\)

Từ (1); (2) suy ra \(A\geq \frac{3}{4}\left(\frac{1}{x}+\frac{1}{y}+\frac{1}{z}-1\right)\)

Mà theo BĐT Cauchy- Schwarz ta có:

\(\frac{1}{x}+\frac{1}{y}+\frac{1}{z}\geq \frac{9}{x+y+z}=\frac{9}{3}=3\)

Do đó: \(A\geq \frac{3}{4}(3-1)=\frac{3}{2}\) (đpcm)

Dấu bằng xảy ra khi \(x=y=z=1\)

14 tháng 4 2022

Sửa đề: \(\dfrac{x}{x+1}+\dfrac{y}{y+1}+\dfrac{z}{z+1}\ge\dfrac{3}{4}\)

Đặt \(P=\dfrac{x}{x+1}+\dfrac{y}{y+1}+\dfrac{z}{z+1}\)

\(P=\dfrac{x+1}{x+1}-\dfrac{1}{x+1}+\dfrac{y+1}{y+1}-\dfrac{1}{y+1}+\dfrac{z+1}{z+1}-\dfrac{1}{z+1}\)

\(P=1-\dfrac{1}{x+1}+1-\dfrac{1}{y+1}+1-\dfrac{1}{z+1}\)

\(P=3-\left(\dfrac{1}{x+1}+\dfrac{1}{y+1}+\dfrac{1}{z+1}\right)\)

Ta có:

\(\dfrac{1}{x+1}+\dfrac{1}{y+1}+\dfrac{1}{z+1}\ge\dfrac{9}{x+y+z+3}\)

\(\Leftrightarrow\dfrac{1}{x+1}+\dfrac{1}{y+1}+\dfrac{1}{z+1}\ge\dfrac{9}{4}\) ( vì \(x+y+z=1\) )

\(\Rightarrow P\ge3-\dfrac{9}{4}=\dfrac{3}{4}\left(đpcm\right)\)

Dấu "=" xảy ra khi \(x+1=y+1=z+1\)

                               \(\Leftrightarrow x=y=z=\dfrac{1}{3}\)

Vậy \(Max_P=\dfrac{3}{4}\) khi \(x=y=z=\dfrac{1}{3}\)

14 tháng 4 2022

thanks bạn

 

18 tháng 2 2022

vỗ tay vì chữ đợp quớ:>

NV
22 tháng 3 2021

\(VT=\dfrac{x^2}{x^2+2xy+3zx}+\dfrac{y^2}{y^2+2yz+3xy}+\dfrac{z^2}{z^2+2zx+3yz}\)

\(VT\ge\dfrac{\left(x+y+z\right)^2}{x^2+y^2+z^2+5xy+5yz+5zx}=\dfrac{\left(x+y+z\right)^2}{\left(x+y+z\right)^2+3\left(xy+yz+zx\right)}\ge\dfrac{\left(x+y+z\right)^2}{\left(x+y+z\right)^2+\left(x+y+z\right)^2}=\dfrac{1}{2}\)

15 tháng 4 2018

\(\dfrac{1}{1+x^2}+\dfrac{1}{1+y^2}\ge\dfrac{2}{1+xy}\)

\(\left(\dfrac{1}{1+x^2}-\dfrac{1}{1+xy}\right)+\left(\dfrac{1}{1+y^2}-\dfrac{1}{1+xy}\right)\ge0\)

\(\left(\dfrac{1+xy-\left(1+x^2\right)}{\left(1+x^2\right)\left(1+xy\right)}\right)+\left(\dfrac{1+xy-\left(1+y^2\right)}{\left(1+y^2\right)\left(1+xy\right)}\right)\ge0\)

\(\left(\dfrac{1+xy-1-x^2}{\left(1+x^2\right)\left(1+xy\right)}\right)+\left(\dfrac{1+xy-1-y^2}{\left(1+y^2\right)\left(1+xy\right)}\right)\ge0\)

\(\dfrac{-x\left(x-y\right)}{\left(1+x^2\right)\left(1+xy\right)}+\dfrac{-y\left(y-x\right)}{\left(1+y^2\right)\left(1+xy\right)}\ge0\)

\(\dfrac{-x\left(x-y\right)\left(1+y^2\right)}{\left(1+x^2\right)\left(1+y^2\right)\left(1+xy\right)}+\dfrac{y\left(x-y\right)\left(1+x^2\right)}{\left(1+x^2\right)\left(1+y^2\right)\left(1+xy\right)}\ge0\)

=> -x(x-y)(1+y2)+y(x-y)(1+x2) ≥ 0

⇔ (x-y)[-x(1+y2)+y(1+x2)]≥0

⇔ (x-y)(-x-xy2+y+x2y) ≥0

⇔ (x-y)[-(x-y)+(x2y-y2x)] ≥ 0

⇔ (x-y)[-(x-y)+xy(x-y) ]≥ 0

⇔ (x-y)(x-y)(xy-1)≥ 0

⇔ (x-y)2 (xy-1) ≥0 (luôn đúng ∀ xy ≥ 1)

=> đpcm

22 tháng 3 2019

bạn pải giả sử trước chứ nếu ntn thì người chấm hỏi ai cho lôi phần chứng minh ra làm phần mục đề